0 Daumen
1,5k Aufrufe

 Transformieren Sie den Vektor v=(4,-3,1)^T

mittels Givens–Rotation auf ein Vielfaches des ersten Einheitsvektors (1,0,0)^T .Geben Sie die dabei verwendeten Givens–Matrizen explizit an. 

Was muss man genau hier machen, wäre nett wenn jemand für mich einen Ansatz hätte

Avatar von

1 Antwort

+1 Daumen
 
Beste Antwort

Hi,
mit der Givens Transformation kann man z.B. eine Matrix \( A \) in eine obere Dreiecksmatrix transformieren.
Will man das \( (i,j)-te \) Element der Matrix \( A \) zu Null bekommen, ist die Givens Matrix \( G \) wie folgt definiert.
$$ G_{i,i}= G_{j,j}=c $$ und
$$ G_{i,j}=-G_{j,i}=s $$ und
$$ G_{k,k}=1 $$ für  \( k\ne i \) und \( k\ne j \) die restlichen Element sind \( 0 \)
Die Größen \( c \) und \( s \) sind wie folgt definiert$$ c=\frac{a_{j,j}}{\sqrt{a_{j,j}^2+a_{i,j}^2}} $$ und
$$ s=\frac{a_{i,j}}{\sqrt{a_{j,j}^2+a_{i,j}^2}} $$
Zuerst will man das \( (2,1) \) Element des Vektors \( x=\begin{pmatrix} 4\\-3\\1 \end{pmatrix} \)zu Null bekommen und danach das \( (3,1) \) Element des resultierenden Vektors. Also lautet die Transformation
$$ G_{3,1}G_{2,1}x $$ mit
$$ G_{2,1}=\begin{pmatrix}  c & s & 0 \\ -s & c & 0 \\ 0 & 0 & 1 \end{pmatrix} = \begin{pmatrix}  0.8 & -0.6 & 0 \\ 0.6 & 0.8 & 0 \\ 0 & 0 & 1 \end{pmatrix} $$ und
$$ G_{3,1}=\begin{pmatrix}  c & 0 & s \\ 0 & 1 & 0 \\ -s & 0 & c \end{pmatrix}=\begin{pmatrix}  0.981 & 0 & 0.196 \\ 0 & 1 & 0 \\ -0.196 & 0 & 0.981 \end{pmatrix} $$
Das ergibt dann den transformierten Vektor
$$ \begin{pmatrix} 5.099\\0\\0 \end{pmatrix} $$

Avatar von 39 k

Aber hast du da kein Fehler gemacht

Muss man nicht G21 nicht erst mit dem Vektor x multiplizieren und dann erst eliminieren??


Oder gilt das nur für Matrizen und nicht für Vektoren aber eigentlich darf da es keine Unterschiede geben oder

Hi, verstehe Deinen Kommentar nicht ganz. Ich habe zuerst \( G_{2,1} \) basierend auf dem Ausgangsvektor \( \begin{pmatrix} 4 \\ -3 \\ 1 \end{pmatrix} \) ausgerechnet und komme auf folgendes \( G_{2,1}x =  \begin{pmatrix} 5 \\ 0 \\ 1 \end{pmatrix} \)

Danach habe ich \( G_{3,1} \) basierend auf \( \begin{pmatrix} 5 \\ 0 \\ 1 \end{pmatrix} \) ausgerechnet und komme dann auf das Endergebnis.

Ahso ja ok aber woher weißt du wie g31 aussieht ?? Woher weiß du das g31 so aussehen muss ?

Hi,

Du willst das \( (3,1) \) Element zu Null machen, also gilt \( i=3 \) und \( j =1 \). Damit ist \( G_{3,3} = G_{1,1} = c \) und \( G_{3,1} = -G_{1,3} = s \) und es gilt

$$ c = \frac{a_{1,1}}{\sqrt{a_{1,1}^2+a_{1,3}^2}} = \frac{5}{\sqrt{25+1}} = 0.981 $$ \( s \) berechnest Du auf die selbe Art, dann kommst Du auf die angegebene Matrix.

Ok Hab es verstanden, weißt du Vielleicht auch wie man eine Probe machen kann damit man weiß ob es richtig ist ??

Was meinst Du mit Probe? Bei richtiger Berechnung der Givens Matrix  und anschließender Multiplikation mit dem entsprechenden Vektor sollte die gewünschte Komponente Null werden. In dem Beispiel ist das ja auch so. Einmal kommt $$ \begin{pmatrix} 5\\0\\1 \end{pmatrix} $$ heraus und dann das vielfache des Einheitsvektors. Es werden also genau die gewünschten Komponenten zu Null.

Ein anderes Problem?

Stell deine Frage

Willkommen bei der Mathelounge! Stell deine Frage einfach und kostenlos

x
Made by a lovely community